Đến nội dung

canhhoang30011999 nội dung

Có 278 mục bởi canhhoang30011999 (Tìm giới hạn từ 03-06-2020)



Sắp theo                Sắp xếp  

#638920 Marathon số học Olympic

Đã gửi bởi canhhoang30011999 on 08-06-2016 - 14:21 trong Số học

Đây là ý kiến của mình từ lâu rồi không phải nói bài này đâu nhé. Do topic có mục đích giao lưu học hỏi là chính nên ở 1 số bài toán nếu người đề xuất có khác với các cách được đưa ra thì cũng nên post lời giải của mình lên.

p/s bài 36 anh imoer có thể post đáp án được không ạ




#638840 Marathon số học Olympic

Đã gửi bởi canhhoang30011999 on 08-06-2016 - 07:56 trong Số học

Hì hì, ở đây có thể dùng Cyclotomic Polynomial để chứng minh sơ cấp Dirichlet, nhưng mình nghĩ dùng 1 định lý thậm chí còn mạnh hơn cả bài này để giải nó thì hơi quá.

Nhân tiện bạn có thể nói sơ qua ý tưởng được không :D

Thực ra nguyên lí Đi-rich-lê chứng minh rất phức tạp nhưng theo mình biết thì khi thi cử không cần cm nên thấy cũng nhiều bài toán phải sử dụng luôn chứ cũng khồng có gì quá cả

p/s bài 50 Ego ơi




#638465 Marathon số học Olympic

Đã gửi bởi canhhoang30011999 on 06-06-2016 - 10:02 trong Số học

.

Với $y\ge 2$ thì $3^y+4\equiv 4\ (\bmod{9})$ nên $7^x\equiv 4\ (\bmod{9})$. Từ đó suy ra: $x\equiv 2\ (\bmod{4})$, hay $x=2k$, với 

Theo em thì đoạn này phải là $x\equiv 2(mod3)$ chứ (vì mà là $ord_{9}(7)$)

đây là lời giải của mình

xét $y=1$ thì suy ra $x=1$.

xét $x=1$ thì suy ra $y=1$.

xét $x,y>1$ thì $7/ 3^{y-1}-1$ nên $6/y-1$ ($ord_{7}(3)=6$)

nên $(3^{6}-1/ 3(3^{y-1}-1)=7(7^{x-1}-1)$ nên $13/7^{x-1}-1$

nên $12/ x-1$ (do $12=ord_{13}(7)$) (vô lý do $x \equiv 2 (mod 3)$




#637281 Marathon số học Olympic

Đã gửi bởi canhhoang30011999 on 31-05-2016 - 23:41 trong Số học

Lời giải của anh thì em không có ý kiến gì nhưng nó có thể sơ cấp hơn chỉ dùng công thức tập hợp . Còn rõ ràng lời giải này ngắn gọn và hay . Nhưng dù sao vẫn khuyến khích mọi người học cách này . 

P/s : Theo em nghĩ có giải tích số học cũng hay vì nó khá đẹp và cũng dễ hiểu , nếu học lên cao cái này cũng tốt . Đây là ý kiến cá nhân mong m.n tập trung vào topic và không delete post này .

Bằng có thể đăng cách kia không để mọi người tham khảo?




#642234 Marathon số học Olympic

Đã gửi bởi canhhoang30011999 on 26-06-2016 - 10:03 trong Số học

Lời giải bài 61:

Gọi các số nguyên tố bé hơn 2016 lần lượt là $p_{1},p_{2},...p_{k}$ 

theo nguyên tắc Đi-rich-lê thì tồn tại vô số số nguyên tố dạng $2p_{1}p_{2}..p_{k}t +p_{2}p_{3}..p_{k}+2$ và vô hạn số nguyên tố dạng $2p_{1}p_{2}..p_{k}t +p_{2}^{2}p_{3}..p_{k}+2$  

Khi đó tồn tại vô số $p$ là 1 số nguyên tố bất kì là 1 trong 2 dạng trên thỏa mãn $\frac{p-1}{2}$ lẻ

theo định lí Trung Quốc về phần dư thì tồn tại n sao cho

$n\equiv 0 (\bmod p), \\ n\equiv 0(mod \frac{p-1}{2})\\ n\equiv 1(mod 2016!)$

Khi đó $a^{n} \equiv +1,-1 (mod p)$

nên $p/d-3$ hoặc $d-1$ hoặc $d+1$ hoặc $d+3$

do có vô số $p$ nên $d=1$ hoặc $d=3$ hoặc $d=-1$ (loại) hoặc $d=-3$ (loại)

nếu d=1 thì theo Đi-rich-lê thì tồn tại vô số nguyên tố q dạng $p_{1}p_{2}..p{k}t+1$ khi đó chọn $n=2q-1$ thì ta có $a+b+c-1$ chia hết cho $q$

$=>a+b+c=1$ (vô lý)

Nếu $d=3$ thì chọn $n=4q-3$ thì thì $a+b+c-3$ chia hết cho $q$ nên $a+b+c=3 $ do đó $a=b=c=1$




#642244 Marathon số học Olympic

Đã gửi bởi canhhoang30011999 on 26-06-2016 - 11:00 trong Số học

Bài 62 Tìm a,b,c nguyên dương sao cho

$a^{2}+b^{2}+c^{2}$ chia hết cho $2013(ab+bc+ca)$

nguồn (Iran TST 2013)

P/s:Anh IMOer có thể đăng cách giải bài 61 của anh được không ạ




#652946 Marathon số học Olympic

Đã gửi bởi canhhoang30011999 on 05-09-2016 - 21:55 trong Số học

Ở đây hình như anh chưa chỉ ra sự vô hạn

anh quên mất ta có $p-1$ chia hết cho $2^{k}$ nên $p \geq 2^{k}$ nên có vô hạn $p$




#643797 Marathon số học Olympic

Đã gửi bởi canhhoang30011999 on 05-07-2016 - 22:32 trong Số học

Bài 68. Cho $y \in N^{*}$ Chứng minh rằng tồn tại vô hạn các số nguyên tố $p$ thỏa mãn $4/p+1$ và $p$ là ước của 1 số có dạng $2^{n}y+1$ (n nguyên dương)

p/s anh IMOer post lời giải bài 64 được không ạ?




#643792 Marathon số học Olympic

Đã gửi bởi canhhoang30011999 on 05-07-2016 - 22:11 trong Số học

Bài 67  (bài này có trong sách của mình)

ta có phương trình Fec-ma $a^{4}+b^{4}=c^{2}$ và $a^{4}+b^{4}=2c^{2}$ chỉ có nghiệm tầm thường nên $x^{2^{k}}+y^{2^{k}}$ không là số chính phương cũng không là 2 lần số chính phương 

Từ đó ta có tồn tại $p$ lẻ sao cho $v_{p}(x^{2^{k}}+y^{2^{k}})$ lẻ

khi đó ta có $x^{2^{k+1}}-y^{2^{k+1}} \equiv 0 (\bmod p)$

suy ra tồn tại a sao cho $ a^{2^{k+1}} \equiv 1 (\bmod p)$

từ đó dễ thấy $2^{k+1}$ là cấp của $a$ mod $p$ từ đó ta có $p-1$ chia hết cho $2^{k+1}$

$=> v_{p}(x^{p-1}-y^{p-1})=v_{p}(x^{2^{k+1}}-y^{2^{k+1}})=v_{p}(x^{2^{k}}+y^{2^{k}})$ lẻ (đpcm)




#642440 Marathon số học Olympic

Đã gửi bởi canhhoang30011999 on 27-06-2016 - 14:58 trong Số học

Anh làm hơi vội đoạn này rồi ạ ví dụ $p=61$ và $d=12$

Chỗ đó chỉ suy ra $v_2{(d)}=v_2{(p-1)}$ thôi ạ

có thể bạn nhầm đề nhưng $p=4^{n}+1$ mà




#636835 Marathon số học Olympic

Đã gửi bởi canhhoang30011999 on 30-05-2016 - 16:35 trong Số học

Lời nhắn tới bạn canhhoang30111999

không thể có Th đó được vì mình đang xét th 

$p^{t}=(ax+by)^{2}+(ay-bx)^{2}=(ax-by)^{2}+(ay-bx)^{2}$ nên nếu chúng có ước nguyên tố chung thì phải là p




#636714 Marathon số học Olympic

Đã gửi bởi canhhoang30011999 on 30-05-2016 - 00:29 trong Số học

Chứng minh $ac+bd,ad-bc$ nguyên tố cùng nhau đi bạn .

đầu tiên ta sẽ chứng minh $p^{t}=x^{2}+y^{2}$ với $(x,y)=1$ với $p=4k+1$

ta cm bằng quy nạp dễ thấy đúng với t=1

giả sử đúng đến t ta sẽ cm đúng với t+1

ta có $p^{t+1}=p^{t}.p=(x^{2}+y^{2})(a^{2}+b^{2})=(ax+by)^{2}+(ay-bx)^{2}=(ax-by)^{2}+(ay+bx)^{2}$

nên nếu $p^{t+1}$ không viết đc dưới dạng tổng của 2 số chính phương nguyên tố cùng nhau thì $p/ax+by,ay-bx,ax-by,ay+bx$

mà $a,b<p$ nên $p/x$,$p/y$ (vô lý)

vậy $p^{t}=x^{2}+y^{2}$ với $(x,y)=1$

ta có với n chỉ gồm các ước nguyên tố dạng $4k+1$ thì $n=(a_{1}^{2}+b_{1}^{2})....(a_{k}^{2}+b_{k}^{2})$ với $(a_{i},b_{i})=1,(a_{i}^{2}+b_{i}^{2})=1$

ta chỉ cần cm $(a^{2}+b^{2})(x^{2}+y^{2})$ phân tích đc thành 2 SCP nguyên tố cùng nhau rồi sau đó quy nạp là xong

hay là cm $(ax+by,ay-bx)=1$

gọi $(ax+by,ay-bx)=d$ thì $b(x^{2}+y^{2})$ chia hết cho p là ước nguyên tố của d

nếu b $b$ chia hết cho $p$ thì $ax$,$ay$ chia hết cho p nên vô lý

vậy $x^{2}+y^{2}$ chia hết cho p

tương tự $a^{2}+b^{2}$ chia hết cho p (vô lý) ta có đpcm 

p/s đang đánh dở mới nhận thông báo có bài mới nên nếu có giống bài trên mong mọi người thông cảm




#636543 Marathon số học Olympic

Đã gửi bởi canhhoang30011999 on 29-05-2016 - 15:43 trong Số học

Lời giải bài 17.

ta có xét đa thức $P(x)$ hệ số nguyên bất kì thì tồn tại vô số nguyên tố $p_{k}$ sao cho tồn tại $n_{k}$ sao cho $P(n_{k})$ chia hết cho $p_{k}$

ta có $P(x)$ và $P'(x)$ nguyên tố cùng nhau nên tồn tại $g(x),f(x)$ hệ số nguyên và n nguyên sao cho

$P(x)g(x)+P'(x)f(x)=n$ với mọi $x$ nguyên

ta xét k số nguyên tố $p_{1}$,...,$p_{k}$ sao cho $p_{i}>n$ và tồn tại $n_{i}$ sao cho $P(n_{i})$ chia hết cho $p_{i}$ 

TH1 nếu $P(n_{i})$ không chia hết cho $p_{i}^{2}$ với mọi $1\leq i \leq k$ thì ta xét n sao cho $n\equiv n_{i} (mod p_{i}^{2})$ với mọi $1\leq i \leq k$ có n thỏa đề ra

TH2 nếu tồn tại $i$ sao cho $P(n_{i})$ chia hết cho $p_{i}^{2}$ thì ta xét

$P(n_{i}+p_{i})=\sum a_t(n_{i}+p_{i})^{k}\equiv P(n_{i})+p_{i}P'(n_{i})(mod p_{i}^{2})$

 nên $P(n_{i}+p_{i})$ chia hết cho $p_{i}$ nhưng không chia hết cho $p_{i}^{2}$ rồi làm như TH1

Bài 18.  giải pt nghiệm nguyên $x^{2}+5=y^{3}$

nguồn Các phương pháp giải toán qua các kì thi olympic 2014




#632372 Inequalities From 2016 Mathematical Olympiads

Đã gửi bởi canhhoang30011999 on 10-05-2016 - 22:12 trong Bất đẳng thức - Cực trị

Bài 31 (Iran TST). Cho bốn số thực dương $a,b,c,d$ thỏa mãn điều kiện $\frac{1}{a+1}+\frac{1}{b+1}+\frac{1}{c+1}+\frac{1}{d+1}=2.$ Chứng minh rằng
$$\sqrt{\frac{a^2+1}{2}} +\sqrt{\frac{b^2+1}{2}}+\sqrt{\frac{c^2+1}{2}}+\sqrt{\frac{d^2+1}{2}} +8 \geqslant 3(\sqrt{a}+\sqrt{b}+\sqrt{c}+\sqrt{d}).$$

Bài 32. (Kyiv Mathematical Festival)

1. Cho ba số thực không âm $a,b,c$ thỏa mãn điều kiện $ab+bc+ca=2.$ Chứng minh rằng \[\frac{ab}{c+1}+\frac{bc}{a+1}+\frac{ca}{b+1}+2(a+b+c) \geqslant 6.\]
2. Cho ba số thực không âm $a,b,c$ thỏa mãn điều kiện $ab+bc+ca=3.$ Chứng minh rằng \[\frac{ab}{c+1}+\frac{bc}{a+1}+\frac{ca}{b+1} \geqslant \frac{3}{2}.\]


 

31 

ta sẽ cm $\sqrt{\frac{a^{2}+1}{2}}-3\sqrt{a}\geq \frac{4}{a+1}-4$

$\Leftrightarrow \frac{(a+1)(a-1)^{2}}{2(\sqrt{\frac{a^{2}+1}{2}}+\sqrt{a})}-2\sqrt{a}(\sqrt{a}-1)^{2}\geq 0$

do $\sqrt{\frac{a^{2}+1}{2}}+\sqrt{a}\leq a+1$

nên ta cần cm $(\sqrt{a}-1)^{2}(\frac{(\sqrt{a}+1)^{2}}{2}-2\sqrt{a})\geq 0$(luôn đúng)

thiết lập các bất đẳng thức tương tự ta có đpcm

p\s có ai làm được bài Nga chưa?




#631751 Inequalities From 2016 Mathematical Olympiads

Đã gửi bởi canhhoang30011999 on 07-05-2016 - 15:14 trong Bất đẳng thức - Cực trị

Bài 25 (Croatia TST). Cho số nguyên $n \geqslant 1$ và $x_1,x_2, \ldots, x_n$ là các số thực không âm. Chứng minh rằng \[\left(x_1 + \frac{x_2}{2} + \cdots + \frac{x_n}{n}\right)\left(x_1 + 2x_2 + \cdots + nx_n\right) \leqslant \frac{(n+1)^2}{4n} (x_1 + x_2 + \cdots + x_n)^2.\]
 

ta có $(x_{1}+2x_{2}+...+nx_{n})(nx_{1}+\frac{n}{2}x_{2}+...+x_{n})\leq \frac{((n+1)x_{1}+(\frac{n}{2}+2)x_{2}+...+(n+1)x_{n})^{2}}{4}$

$\frac{\leq (n+1)^{2}(x_{1}+x_{2}+...+x_{n})^{2}}{4}$ (dpcm)

dấu bằng xảy ra khi $x_1=x_n=1,x_{2}=x_{3}=...=x_{n-1}=0$ 




#629403 Inequalities From 2016 Mathematical Olympiads

Đã gửi bởi canhhoang30011999 on 24-04-2016 - 20:35 trong Bất đẳng thức - Cực trị

Bài 23 (Romania JBMO TST 2016). Với $m,n$ là hai số tự nhiên và ba số thực $x,y,z$ thuộc $[0,1].$ Chứng minh rằng
\[0 \leqslant x^{m+n}+y^{m+n}+z^{m+n}-x^my^n-y^mz^n-z^mx^n \leqslant 1.\]
Đẳng thức xảy ra khi nào ?

 

Bài 24 (China Junior High School Mathematics League). Cho ba số thực $x,y$ và $z$ thỏa mãn điều kiện $x+y+z=1.$ Tìm giá trị lớn nhất của biểu thức $xy+2yz+3zx.$
 

24 ta chứng minh $xy+2yz+3zx \leq \frac{3} {4}$

tức là chứng minh $(x+y+z)^{2}\geq \frac{4}{3}(xy+2yz+3zx)$

hay $x^{2} +x(\frac{2}{3}y-2z)+y^{2}+z^{2}-\frac{2}{3}yz \geq 0$ với mọi $x$ thuộc R

tức là $\bigtriangleup \leq 0$

hay $\frac{-32}{9}y^{2} \leq 0$(luôn đúng)

dấu bằng xảy ra khi $x=z=\frac{1}{2},y=0$




#637865 Cập nhật tình hình, thảo luận, chém gió về kì thi vào lớp 10 THPT

Đã gửi bởi canhhoang30011999 on 03-06-2016 - 19:54 trong Góc giao lưu

Tuần sau em thi mà giờ vẫn chưa ôn văn được mấy,toán với anh thì đã max gà rồi chắc xác định quá :(

Có bạn nào ở Bình Dương không,xem đề tốt nghiệp Tiếng anh của Bình Dương thấy kinh quá  :(

Ps:Chia sẻ vậy thui chứ giờ phải off để ôn văn nữa .....

Tiếng Anh với Văn thì em không phải lo đâu quan trọng là toán thôi (Tiếng Anh anh 4 năm 7, mà thi vẫn được 9.5)




#523232 $\boxed{TOPIC}$ Véc-tơ và ứng dụng

Đã gửi bởi canhhoang30011999 on 07-09-2014 - 10:28 trong Hình học phẳng

 

$21)$ Cho tam giác $ABC$; trọng tâm $G$. $I$ là trung điểm $AG$; $J$ là trọng tâm tam giác $BIG$.

Yêu cầu biểu diễn các vecto $\overrightarrow{BC};\overrightarrow{AG};\overrightarrow{BI};\overrightarrow{BJ};\overrightarrow{AJ}$ theo $\overrightarrow{AB};\overrightarrow{AC}$

 

21 ta có $\overrightarrow{BC}=\overrightarrow{AC}-\overrightarrow{AB}$

$3\overrightarrow{AG}= \overrightarrow{AB}+\overrightarrow{BG}+\overrightarrow{AC}+\overrightarrow{CG}+\overrightarrow{AG}$

$\Rightarrow 3\overrightarrow{AG}= \overrightarrow{AB}+\overrightarrow{AC}$

tương tự 

$3\overrightarrow{BG}= \overrightarrow{BA}+\overrightarrow{BC}= \overrightarrow{AC}-2\overrightarrow{AB}$

$\overrightarrow{BI}= \frac{1}{2}(\overrightarrow{BG}-\overrightarrow{AB})= \frac{1}{6}(2\overrightarrow{AC}-\overrightarrow{AB})$

tương tự ta tìm được BJ,AJ




#631237 Thông tin về VMF trên Alexa

Đã gửi bởi canhhoang30011999 on 04-05-2016 - 20:34 trong Diễn đàn Toán học trên chặng đường phát triển

thầy có thể đăng của mathscope để tham khảo đc không ạ




#486956 Trận 4 - Bất đẳng thức

Đã gửi bởi canhhoang30011999 on 15-03-2014 - 16:20 trong Thi giải toán Marathon cấp THCS 2014

Bài làm của MSS 34:

 

Áp dụng bđt $4ab\leq (a+b)^{2}$ với mọi $a,b$:

$4xy\leq (x+y)^{2}$$\Leftrightarrow 2\leq (x+y)^{3}+4xy\leq (x+y)^{3}+(x+y)^{2}$

Đặt $x+y=t$

Bpt trở thành: $t^{3}+t^{2}-2\geq 0$

$\Leftrightarrow t^{3}-t^{2}+2t^{2}-2\geq 0$

$\Leftrightarrow t^{2}(t-1)+2(t-1)(t+1)\geq 0$

$\Leftrightarrow (t-1)(t^{2}+2t+2)\geq 0$

$\Leftrightarrow (t-1)\geq 0$ (do $t^{2}+2t+2=t^{2}+2t+1+1=(t+1)^{2}+1> 0$ mọi $t$ )

$\Leftrightarrow t\geq 1$

$\Rightarrow x+y\geq 1$

$\Leftrightarrow (x+y)^{2}\geq 1$

mà $2x^{2}+2y^{2}\geq (x+y)^{2}$ với mọi $x,y$

$\Leftrightarrow 2x^{2}+2y^{2}\geq 1$

$\Leftrightarrow x^{2}+y^{2}\geq \frac{1}{2}$

Ta có:$A=3(x^{4}+y^{4}+x^{2}y^{2})-2(x^{2}+y^{2})+1= 3(x^{2}+y^{2})^{2}-2(x^{2}+y^{2})-3x^{2}y^{2}+1$

Áp dụng bđt $ab\leq \frac{(a^{2}+b^{2})}{2}$ với mọi $a,b$

$3x^{2}y^{2}\leq \frac{3(x^{2}+y^{2})^{2}}{4}$

nên $3(x^{2}+y^{2})^{2}-2(x^{2}+y^{2})-3x^{2}y^{2}+1\geq 3(x^{2}+y^{2})^{2}-2(x^{2}+y^{2})-\frac{3(x^{2}+y^{2})^{2}}{4}+1=\frac{9}{4}(x^{2}+y^{2})^{2}-2(x^{2}+y^{2})+1=\frac{9}{4}(x^{2}+y^{2})^{2}-\frac{9}{16}-2(x^{2}+y^{2})+1+\frac{9}{16}=\frac{9}{4}(x^{2}+y^{2}-\frac{1}{2})(x^{2}+y^{2}+\frac{1}{2})-2(x^{2}+y^{2}-\frac{1}{2})+\frac{9}{16}=(x^{2}+y^{2}-\frac{1}{2})(\frac{9}{4}x^{2}+\frac{9}{4}y^{2}-\frac{7}{8})+\frac{9}{16}\geq \frac{9}{16}$

 (do $x^{2}+y^{2}\geq \frac{1}{2}$)

Vậy $Min A =\frac{9}{16}\Leftrightarrow x^{2}+y^{2}= \frac{1}{2}$ và $x=y$$\Leftrightarrow x=y=\frac{1}{2}$

 

Điểm 10 .

Đoạn đấu = bạn thiếu điều kiên x+y=1 (thiếu điều kiện này bạn không thể suy ra $x= y= \frac{1}{2}$ được )

Đoạn tô màu đỏ là bất đăng thức BCS




#635854 Marathon Tổ hợp và rời rạc VMF

Đã gửi bởi canhhoang30011999 on 27-05-2016 - 08:32 trong Tổ hợp và rời rạc

Lời giải bài 7 : 


Ta có $T_{4}=T_{5}=T_{6}=6$

Xét với $n>6$ ta có số bộ $(1,a_{2},...,a_{n},1)$ thỏa mãn đk $1,2,3$ bằng số bộ $(1,a_{2},....,a_{n-2},a_{n-1},1)$ với $a_{n-2}$ khác 1 thỏa 1,2,3 cộng với số bộ

$(1,a_{2},....,1,a_{n-1},a{n},1)$ thỏa đk $1,2,3$

nên $T_{n+1}=T{n}+4T_{n-2}$ với $n\geq 6$ (công thức tổng quát phải dùng số phức lằng nhằng nên mình không post)

còn ý b thì chọn $g_{n}=T_{n}$ là được mà

Bài 8 : (Macedonia TST $2016$)(bài này bữa post rồi mà chưa thấy ai giải nên mình cho vào đây không biết có phạm quy không)

. Cho lưới vuông kích thước 2n×2n, chứa các ô vuông đơn vị trắng. Trong một nước đi một người có thể đổi màu của ba ô liên tiếp trong cùng một hàng hoặc một cột, với quy ước trắng thành đen và đen thành trắng. Xác định tất cả số nguyên dương $n\geq 2$, sao cho với hữu hạn nước đi, ta có thể thu được một bàn cờ vua.




#635950 Marathon Tổ hợp và rời rạc VMF

Đã gửi bởi canhhoang30011999 on 27-05-2016 - 15:23 trong Tổ hợp và rời rạc

Bài 8 hình như lời giải của JUV có vấn đề vì xét trường hợp 2 ô đen 1 ô trắng sẽ chuyển thành 2 ô trắng 1 ô đen thì sẽ không đồng dư với nhau (mod 3) 




#635885 Marathon Tổ hợp và rời rạc VMF

Đã gửi bởi canhhoang30011999 on 27-05-2016 - 10:10 trong Tổ hợp và rời rạc

Lời giải bài 7:

Ta đánh số 1,2,3 cho các ô sao cho ô ở góc trái trên cùng đánh số 1, các ô nằm kề phía trái ô 1 đánh số 2, ô nằm kề trái ô 2 đánh số 3, ô nằm kề trái ô 3 đánh số 1. Nếu mà ô cuối cùng của hàng này đánh 1 trong các số 1,2,3 thì ô đầu tiên ở hàng dưới đánh các số lần lượt là 2,3,1. Nếu $n$ là bội của 3 thì các hàng của bảng đều được đánh cùng 1 số. Xét $n=3$ thấy thoả mãn, trường hợp $n=3k$ thì ta có thể chia bảng ra làm $k^2$ ô vuông $3\times 3$ và thay đổi các bảng đấy sao cho bảng to trở thành 1 bàn cờ vua. Xét $n$ không là bội của 3, lúc đó trong 3 ô liên tiếp bất kì đều chứa cả 3 số 1,2,3. Gọi $a_t;b_t;c_t$ lần lượt là số ô đen được ghi số 1,2,3 sau nước đi thứ $t$. Ta thấy rằng trong 1 nước đi, số các ô đen được đánh số 1,2,3 hoặc giảm 1 hoặc tăng thêm 1, vì vậy $a_{t+1};b_{t+1};c_{t+1}$ luôn lần lượt khác tính chẵn lẻ với $a_t;b_t;c_t$. Vì lúc đầu có 0 ô đen được đánh số 1,2,3 nên $a_t;b_t;c_t$ luôn cùng tính chẵn lẻ với nhau với mọi $t$ tự nhiên. Tuy nhiên khi thu được 1 bàn cờ vua thì số các ô đen đánh số 1,2,3 không đôi một cùng tính chẵn lẻ nên không thể thu được 1 bàn cờ vua

 

bạn xem lại đoạn cuối cái (xét bảng 8*8 thì có 12 số 1 10 số 3 và 10 số 2)




#635866 Marathon Tổ hợp và rời rạc VMF

Đã gửi bởi canhhoang30011999 on 27-05-2016 - 09:12 trong Tổ hợp và rời rạc

Lời giải bài 5:

Chọn ra 2 số thuộc $a,b$ thuộc $S$ mà $a,b$ phân biệt mà $2a\leq n< 2b$. Xét 1 cặp $(a;b)$ như thế, ta thấy rằng chỉ có 1 cấp số cộng cực đại duy nhất chứa 2 số $a$ và $b$ đó mà không chứa bất kì số nào nằm giữa 2 số đó. Tương tự, 2 cấp số cộng cực đại công sai khác 0 mà có chứa 2 số (a;b) thoả mãn  $2a< n\leq 2b$ và không chứa bất kì số nào nằm giữa 2 số đó thì sẽ trùng nhau. Vì vậy có thể thiết lập 1 song ánh giữa tập các cặp $(a;b)$ vào tập các csc cực đại công sai khác 0.Mà có thể tạo được $n$ csc cực đại công sai =0 nên số các csc cực đại bằng số cách chọn cặp $(a;b)$ cộng thêm $n$ và sẽ bằng : $(\frac{n}{2})(\frac{n}{2})+n$ nếu $n$ chẵn và bằng $(\frac{n-1}{2}+1)(\frac{n-1}{2})+n$ nếu $n$ lẻ.

 

hình như bạn bị thừa TH ví dụ với $n=10$ thì theo bạn có $1$ cấp số cộng cực đại ứng với $2,8$ nhưng thực ra csc này không cực đại vì ta có thể thêm $5$vào để thành csc $2,5,8$




#635838 Marathon Tổ hợp và rời rạc VMF

Đã gửi bởi canhhoang30011999 on 27-05-2016 - 00:55 trong Tổ hợp và rời rạc

Mình đọc cả $2$ cách thì thấy cách của JUV bị thừa mất $1$ số TH CSC chỉ có $2$ số nhưng mà mình thấy với $n$ đủ lớn thì đáp số của Long Phi phải lớn hơn của JUV không biết $2$ bạn ai có nhầm ở đâu không(mình đọc thấy đúng cả)?